4
$\begingroup$

Let $K$ be a field and $\alpha_i$ ($i=1,\dots,n$) be Galois conjugates. Let $L=K(\alpha_1,\dots,\alpha_n)$ and $G=Gal(L/K)$. We embed $G$ in $S_n$ by its action on $\alpha_i$. Let $H$ be another subgroup of $S_n$, acting on the variables $X_i$ ($i=1,\dots,n$). Let $R=K[X_1,\dots,X_n]^H$ be the ring of invariant polynomials under $H$. Suppose for all $f\in R$, $f(\alpha_1,\dots,\alpha_n)\in K$. Does it follow that $G$ is contained in $H$?

Note that when $H=A_n$ we can take $f$ to be the discriminant, and this question reduces to the equivalence between the discriminant being in $K$ and the Galois group being in $A_n$.

$\endgroup$
1
  • $\begingroup$ Just a note to say that these invariant polynomials $R$ form the basis of the "resolvent method" or the "Stauduhar method" to compute the Galois group of a given polynomial, precisely by using the above result to prove that $G$ is contained in some $H$. The paper "Computation of Galois groups of rational polynomials" by Fieker and Kluners is a good place to read about this. $\endgroup$
    – Doris
    Feb 10, 2017 at 16:00

1 Answer 1

6
$\begingroup$

Lemma 1: Let $H$ be a subgroup of $\mathfrak{S}_n$, and $K$ a field. If $(\xi_1,\ldots,\xi_n)$ and $(\xi'_1,\ldots,\xi'_n)$ are two $n$-tuples of elements of $K$, then t.f.a.e.:

  1. there exists $\sigma\in H$ such that $\xi'_i = \xi_{\sigma(i)}$ for all $i$,

  2. for every $f \in K[X_1,\ldots,X_n]^H$ we have $f(\xi'_1,\ldots,\xi'_n) = f(\xi_1,\ldots,\xi_n)$,

  3. every $f \in K[X_1,\ldots,X_n]^H$ which vanishes at $(\xi_1,\ldots,\xi_n)$ also vanishes at $(\xi'_1,\ldots,\xi'_n)$.

Proof: (1)⇒(2) and (2)⇒(3) are trivial. Assume (1) does not hold: then one can construct a polynomial $h \in K[X_1,\ldots,X_n]$ which vanishes at $(\xi_1,\ldots,\xi_n)$ but at none of the $(\xi'_{\sigma(1)},\ldots,\xi'_{\sigma(n)})$; then $f = \prod_{\sigma\in H} h(X_{\sigma(1)},\ldots,X_{\sigma(n)})$ is in $K[X_1,\ldots,X_n]^H$, vanishes at $(\xi_1,\ldots,\xi_n)$ but not at $(\xi'_1,\ldots,\xi'_n)$, so (3) does not hold.

Lemma 2: Let $H$ be a subgroup of $\mathfrak{S}_n$, and $K \subseteq L$ a field extension. If $(\xi_1,\ldots,\xi_n)$ and $(\xi'_1,\ldots,\xi'_n)$ are two $n$-tuples of elements of $L$, then t.f.a.e.:

  1. there exists $\sigma\in H$ such that $\xi'_i = \xi_{\sigma(i)}$ for all $i$,

  2. for every $f \in K[X_1,\ldots,X_n]^H$ we have $f(\xi'_1,\ldots,\xi'_n) = f(\xi_1,\ldots,\xi_n)$.

Proof: (1)⇒(2) is again trivial. Assume (1) does not hold: then by the previous lemma (applied to $L$ instead of $K$), there exists $f \in L[X_1,\ldots,X_n]^H$ such that $f(\xi'_1,\ldots,\xi'_n) \neq f(\xi_1,\ldots,\xi_n)$. Write $f$ as a $L$-linear combination of polynomials which are each the sum of the orbit of a monomial under the action of $H$: such polynomials are in $K[X_1,\ldots,X_n]^H$, and at least one of them, call it $g$, must satisfy $g(\xi'_1,\ldots,\xi'_n) \neq g(\xi_1,\ldots,\xi_n)$. So (2) does not hold.


Now in the case of your question, assume there exists $\sigma$ in $G$ which is not in $H$. Define $(\alpha'_1,\ldots,\alpha'_n) = (\sigma(\alpha_1),\ldots,\sigma(\alpha_n))$, which is also $(\alpha_{\sigma(1)},\ldots,\alpha_{\sigma(n)})$ by definition of the inclusion of $G$ in $\mathfrak{S}_n$; note that the $\alpha_i$ being implicitly assumed distinct, $\sigma$ is the only $\tau \in \mathfrak{S}_n$ such that $(\alpha'_1,\ldots,\alpha'_n) = (\alpha_{\tau(1)},\ldots,\alpha_{\tau(n)})$, and it is not in $H$. So by lemma 2, there exists $f \in K[X_1,\ldots,X_n]^H$ such that $f(\alpha'_1,\ldots,\alpha'_n) \neq f(\alpha_1,\ldots,\alpha_n)$, which means exactly $\sigma(f(\alpha_1,\ldots,\alpha_n)) \neq f(\alpha_1,\ldots,\alpha_n)$. This contradicts $f(\alpha_1,\ldots,\alpha_n) \in K$.

$\endgroup$

Your Answer

By clicking “Post Your Answer”, you agree to our terms of service and acknowledge you have read our privacy policy.

Not the answer you're looking for? Browse other questions tagged or ask your own question.